Plan ahead for the next school year. Schedule your class today!

G
Topic
First Poster
Last Poster
Quadrilateral ABCD such that AC+AD=BC+BD
WakeUp   16
N 2 minutes ago by shendrew7
Source: Canadian MO 2012 #3
Let $ABCD$ be a convex quadrilateral and let $P$ be the point of intersection of $AC$ and $BD$. Suppose that $AC+AD=BC+BD$. Prove that the internal angle bisectors of $\angle ACB$, $\angle ADB$ and $\angle APB$ meet at a common point.
16 replies
WakeUp
May 4, 2012
shendrew7
2 minutes ago
No more topics!
Problem3
samithayohan   120
N Jul 20, 2025 by LHE96
Source: IMO 2015 problem 3
Let $ABC$ be an acute triangle with $AB > AC$. Let $\Gamma $ be its circumcircle, $H$ its orthocenter, and $F$ the foot of the altitude from $A$. Let $M$ be the midpoint of $BC$. Let $Q$ be the point on $\Gamma$ such that $\angle HQA = 90^{\circ}$ and let $K$ be the point on $\Gamma$ such that $\angle HKQ = 90^{\circ}$. Assume that the points $A$, $B$, $C$, $K$ and $Q$ are all different and lie on $\Gamma$ in this order.

Prove that the circumcircles of triangles $KQH$ and $FKM$ are tangent to each other.

Proposed by Ukraine
120 replies
samithayohan
Jul 10, 2015
LHE96
Jul 20, 2025
Problem3
G H J
G H BBookmark kLocked kLocked NReply
Source: IMO 2015 problem 3
The post below has been deleted. Click to close.
This post has been deleted. Click here to see post.
Shreyasharma
688 posts
#135
Y by
20 minute solve, took time to find the right inversion.
Note that $Q$, $H$ and $M$ are collinear from reflecting $H$ about $M$ to $H'$ and noting that $AH'$ is a diameter of $(ABC)$. Now consider the inversion with power $AH \cdot HF$, which swaps the following pairs of points:
  • The pairs $(A, F)$, $(B, E)$ and $(C, D)$ where $D$ and $E$ are the feet from $B$ and $C$ to $AC$ and $AB$ respectively.
  • The pair $(Q, M)$, as $\angle HQA = \angle HFM = 90$.
  • The pair $(K, N)$, where $N$ denotes the point on $(DEF)$ such that $\angle HMN = 90$.
Hence we wish to show $MN$ is tangent to $(NAQ)$ - however noting that $MN \parallel AQ$ as they are both perpendicular to $QM$, it follows that it suffices to show $NA = NQ$. This would follow immediately if we could show $2MN = AQ$, yet this is clear from the homothety at $H$ with scale factor $2$ sending $(DEF)$ to $(ABC)$, finishing. $\square$
Z K Y
The post below has been deleted. Click to close.
This post has been deleted. Click here to see post.
Mathandski
777 posts
#136
Y by
I inverted at H
Attachments:
Z K Y
The post below has been deleted. Click to close.
This post has been deleted. Click here to see post.
SomeonesPenguin
129 posts
#137 • 1 Y
Y by zzSpartan
So this took about 10 minutes.

Let $O$ be the circumcenter, $A'$ be the $A$ antipode, $N$ be the midpoint of $AH$ and $T$ lies on the nine-point circle such that $TM\perp HM$.

It is known that $\overline{A'-M-H-Q}$. Now invert at $H$ with radius $\sqrt{-HA\cdot HF}$. It's easy to see that $Q\mapsto M$, $K\mapsto T$ and $A\mapsto F$.

After inversion, it suffices to prove that $MT$ is tangent to $(AQT)$. From middle lines, $ON$ is parallel to $A'Q$, hence $ON\perp AQ$ so $ON$ is the perpendicular bisector of $AQ$. Now notice that $NM$ is a diameter in the nine-point circle so $\angle NTM=\angle TMH=90^\circ$ , or $NT\parallel MH$. Therefore, we get $\overline{T-O-N}$ and since $ON$ is the perpendicular bisector of $AQ$, we get the desired conclusion. $\blacksquare$
Z K Y
The post below has been deleted. Click to close.
This post has been deleted. Click here to see post.
ihatemath123
3496 posts
#138 • 1 Y
Y by OronSH
Invert at $H$.

Let $N^*$ be the minor arc midpoint of arc $B^{*}C^*$ in $(A^*B^*C^*)$. Since $N^*$ and $Q^*$ are antipodal arc midpoints in $(A^*B^*C^*)$, it follows that $\angle N^* K^* Q^* = 90^{\circ}$. We also have $\angle K^* Q^* H = 90^{\circ}$ by definition. It's well known that the uninverted $M$, $H$ and $Q$ are collinear, so $M^*$, $H$ and $Q^*$ are collinear – since $M^*$ lies on $(HB^*C^*)$, it follows that $M^*$ is the foot from $F^*$ to line $HQ^*$.

Now, what we want to show is clearly true: $\overline{Q^*K^*} \parallel \overline{M^* F^*}$ and $M^* K^* = F^* K^*$ since line $N^* K^*$ is the perpendicular bisector of $\overline{M^* F^*}$. Therefore, $(F^* K^* M^* )$ is tangent to line $KQ$.
Z K Y
The post below has been deleted. Click to close.
This post has been deleted. Click here to see post.
iStud
272 posts
#140
Y by
reim's spam lol

Note that it suffices to prove that $\angle{MFK}-\angle{QHK}=\angle{MKQ}=90^\circ+\angle{HKM}$.

Let $O$ be the circumcenter of $\triangle{ABC}$. Define $A',Q'$ to be the antipodes of $A,Q$, respectively .Notice that $Q$ is the $A$-queue point of $\triangle{ABC}$, so it's well known that $\overline{Q,H,M,A'}$. Moreover, $\overline{K,H,Q'}$. Let $J$ be the midpoint of $Q'H$. Note that $AF\cap(ABC)$ is the reflection of $H$ over $BC$ and $A'$ is the reflection of $H$ over $M$. Using Reim's Theorem 2 times, we conclude that $AJFK$ and $MJQK$ are cyclic.

Let $\angle{HQK}=\alpha$ and $\angle{JQM}=\beta$. Note that $J$ must be lying on the mid parallel line of $AQ$ and $A'Q'$ (which passes through $O$), so $\angle{AJQ}=2\angle{JQM}=2\beta$. We have $JO\parallel HQ$, so $\angle{AFK}=\angle{AJK}=\angle{AJO}+\angle{OJK}=\frac{\angle{AJQ}}{2}+\angle{QHK}=90^\circ-\alpha-\beta$, then $\angle{MFK}=90^\circ+\angle{AFK}=180^\circ-\alpha+\beta$. Lastly, using information that $\angle{QHK}=90^\circ-\alpha$, we can have $\angle{MFK}-\angle{QHK}=90+\beta=90^\circ+\angle{JQM}=90^\circ+\angle{JKM}=90^\circ+\angle{HKM}$, as desired. $\blacksquare$

P2 of GOWACO 2021 was definitely much harder than this one :P
Attachments:
Z K Y
The post below has been deleted. Click to close.
This post has been deleted. Click here to see post.
Ilikeminecraft
738 posts
#141
Y by
rename appropriately first($F \to D, Q\to D$).
Invert about $H$ first with radius $-\sqrt{AH\cdot HD}.$ Then, the circumcircle maps onto the 9-point circle, $K$ maps to intersection of 9 point circle and the line perpendicular to $GM$ that passes through $M.$ Clearly, $G\leftrightarrow M$. Problem simplifies to proving that $(AGK^*)$ is tangent to $MK^*.$
Then, reflect $H$ across $K^*$ and $M,$ and they both land on the circumcircle. In fact, they form a rectangle. This finishes.
Z K Y
The post below has been deleted. Click to close.
This post has been deleted. Click here to see post.
cj13609517288
1940 posts
#142 • 1 Y
Y by MS_asdfgzxcvb
This works????????

Let $\ell$ be the line through $M$ perpendicular to $HM$, and let $S$ be the point on $\ell$ such that $(QAS)$ is tangent to $\ell$. Since $QA\perp QM$, $S$ is the projection of the midpoint of $QA$ onto $\ell$. So in complex coordinates with $(ABC)$ as the unit circle,
\[s=m+\frac{a-q}{2}\Longrightarrow 2s-h=2m+a-q-h=-q.\]Therefore, the reflection of $H$ over $S$ is exactly the $Q$-antipode, meaning that $K$ lies on $HS$.

Now consider the negative inversion centered at $H$ swapping $A$ and $F$. Then it also swaps $Q$ and $M$, and $K$ and $S$ because they are collinear with $H$ and also $\ell$ swaps with $(QH)$. So since $(QAS)$ is tangent to $\ell$, we get $(MFK)$ is tangent to $(QH)$. $\blacksquare$
Z K Y
The post below has been deleted. Click to close.
This post has been deleted. Click here to see post.
VideoCake
19 posts
#143
Y by
Solution. Let \(A'\) be the antipode of \(A\) on \(\Gamma\). Observe that \(H, M, A'\) are collinear (as \(M\) is known to be the midpoint of \(HA'\)) and observe that \(Q, H, A'\) are collinear (as \(AQ \perp QH\)). Thus, \(Q, M, H\) are collinear. Let \(D, E\) be the foots of the altitudes from \(B, C\).
Perform an inversion around a circle with center \(H\). Denote \(X^*\) to be the inverted image of \(X\). We get that
  • \(F^*D^*E^*\) is a triangle with orthic triangle \(A^*B^*C^*\) and orthocenter \(H^*\).
  • \(Q^*\) lies on the line \(A^*D^*\) and on the circle \((A^*B^*C^*)\), as \(AQDH\) is cyclic.
  • \(K^*\) is on the perpendicular line to \(H^*Q^*\) through \(Q^*\) and on \((A^*B^*C^*)\), as \(K\) lies on the circle with diameter \(QH\).
  • \(M^*\) is the intersection of \((H^*B^*C^*F^*)\) and \(Q^*M^*\), as line \(BC\) goes to the circle \((H^*B^*C^*)\).
It is enough to show that \((M^*F^*K^*)\) is tangent to \(Q^*K^*\). For simplicity, we remove the \(*\) symbols from the points and solve the remaining problem. Let \(O\) be the circumcenter of \((BMFCH)\).
First, it is known that \(H\) is the incenter of \(\Delta ABC\), and that \(D\), \(E\) and \(F\) are the three excenters of \(\Delta ABC\). This means that \(O\) is the \(A\)-southpole and lies on the circle \((ABC)\). In particular, \(O\) is the midpoint of \(HF\). As \(FM \perp MH\), we have \(OMF\) isosceles. As \(AQ\) is the exterior angle bisector of \(\angle BAC\), we know that \(Q\) is the northpole. Therefore, \(OQ\) is the diameter of \(ABC\), meaning \(QK \perp KO\). Notice that \(KO\) is also the perpendicular bisector of \(MF\), so \(KMF\) is isosceles. Notice that \(FM \perp MQ \perp QK \implies MF \parallel QK\). Thus, \(\angle QKM = \angle FMK = \angle KFM\), and \((MFK)\) is indeed tangent to \(QK\), as desired.
Attachments:
Z K Y
The post below has been deleted. Click to close.
This post has been deleted. Click here to see post.
Retemoeg
64 posts
#144
Y by
Let $AF$ intersect $(O)$ twice at $H'$. Denote $L$ the orthogonal projection of $Q$ onto $AD$. We'll call the midpoints of segments $QL, QA, QH$, $Y, X, I$ resp. Let $Z$ be the center of $(KDM)$. Denote $A'$ the antipodal point of $A$ in $(O)$.

We readily notice that $\angle AQH = \angle AQA' = 90^{\circ}$, thus $Q, H, A'$ are collinear. From a common result: $H, M, A'$ are collinear. Hence all these four points lies on a straight line. We'll show that $\angle MKI = \angle ZKM$, i.e $\angle MKI = \angle KFA = \angle H'KF + \angle KH'F$. A spiral similarity $\phi$ centered at $K$ sends $Q$ to $A$ and $H$ to $A'$, thus:
\[ \angle MKI = \angle MKH + \angle HKI = \angle MKH + \angle QHK = \angle MKH + \angle AA'K = \angle MKH + \angle AH'K \]So we'll aim to show that $\angle MKH = \angle H'KF$.
Observe that $\phi$ also sends $F$ to $Y$, $H'$ to $L$, $M$ to $X$. Thus, $\angle MKH = \angle XKQ$ and $\angle H'KF = \angle LKY$. And to finish off the problem, $X$ is the intersection of the two tangents from $Q$ and $L$ w.r.t $(I)$. Thus $KX$ is the $K$-symmedian w.r.t $\triangle QKL$, hence $\angle QKX = \angle LKY$ and we are done.

Click to reveal hidden text
This post has been edited 2 times. Last edited by Retemoeg, Apr 20, 2025, 5:26 AM
Z K Y
The post below has been deleted. Click to close.
This post has been deleted. Click here to see post.
bjump
1070 posts
#145
Y by
Consider a Negative inversion at $H$ such that the circumcircle of $\triangle ABC$ is mapped to the Nine Point Circle of $\triangle ABC$. $F$ is swapped with $A$ under this inversion. $Q$ is mapped to $M$ and vice versa. $K$ is mapped to the point on $(A'B'C')$ such that $\angle HQ'K' = 90^\circ$. Since $M' \in (HB'C')$ we have $\angle F'M'H = \angle F'M'Q' = \angle M'Q'K' = 90^\circ$, so $Q'K' \parallel M'F'$, Note that the $Q'$-antipode in $(A'B'C')$ is the midpoint of $HF'$, $K'$ lies on the perpendicular bisector of $F'M'$ therefore $Q'K'$ is tangent to $(F'M'K')$. Uninverting gives the desired conclusion.
Z K Y
The post below has been deleted. Click to close.
This post has been deleted. Click here to see post.
fearsum_fyz
91 posts
#146
Y by
Consider the composition $\mathcal{T}$ of inversion centered at $H$ with radius $\sqrt{HA \cdot HF}$ and reflection over $H$.
It is easy to see that $\mathcal{T}$ swaps the circumcircle and nine point circle, sending $A$ to $F$ and $Q$ to $M$.
Let $K^*$ be the image of $K$ under $\mathcal{T}$.
Then $\measuredangle{QMK^*} = \measuredangle{HMK^*} = \measuredangle{HKQ} = 90^{\circ}$, so $AQ \parallel MK^*$. Hence it would suffice to show that $A K^* = Q K^*$.
Let $Q'$ be the antipode of $Q$. By homothety at $H$, $K^*$ is the midpoint of $HQ'$.
By Apollonius's theorem in $\Delta{AQ'H}$ and $\Delta{QQ'H}$, we have
$2 A{K^*}^2 + 2 H{K^*}^2 = AQ'^2 + AH^2$
$2 Q{K^*}^2 + 2 H{K^*}^2 = QQ'^2 + QH^2$
Subtracting and using the Pythagorean theorem, we get
$2 A{K^*}^2 - 2 Q{K^*}^2 = AQ^2 - AQ^2 = 0$
as desired.
Attachments:
This post has been edited 2 times. Last edited by fearsum_fyz, May 17, 2025, 12:08 PM
Reason: fixed punctuation
Z K Y
The post below has been deleted. Click to close.
This post has been deleted. Click here to see post.
zuat.e
96 posts
#147 • 1 Y
Y by Kingsbane2139
Consider the inversion at $H$ with radius $r=\sqrt{AH\cdot AF}$ followed by a reflection w.r.t $H$.

It suffices to show that $K'M$ is tangent to $(AQK')$, where $K'$ is the inverse of $K$.

Let $V,W$ be the reflections of $H$ across $K',M$, so as $AQWV$ is cyclic (because $VH\cdot HK=2K'H\cdot HK=2HQ\cdot HM=HQ\cdot HW$ and $W\in (ABC)$) and $\measuredangle AQW=\measuredangle QWV=90º$, we have $ AQWV$ rectangle, yet clearly $K'$ is center of $\triangle VHW$, from where it is clear that $\triangle K'AQ$ is isosceles, consequently $\measuredangle MK'Q=\measuredangle K'QA=\measuredangle K'AQ$, as desired.
Z K Y
The post below has been deleted. Click to close.
This post has been deleted. Click here to see post.
cursed_tangent1434
737 posts
#148
Y by
As a true config main we refrain from indulging in inversion. We start off by noting the following which allows us to adjust our frame of reference.

Claim : Point $H$ is the $Q-$Humpty Point in $\triangle QBC$.

Proof : Simply note that if the $A-$antipode is $A'$,
\[\measuredangle HBC = \measuredangle CAF = \measuredangle A'AB = \measuredangle A'QB = \measuredangle HQB\]a similar argument shows that $\measuredangle BCH = \measuredangle CQH$ which suffices to show the claim.

Now, we rewrite the problem with $\triangle QBC$ as our reference triangle.
Reference Shift wrote:
Let $\triangle ABC$ be a triangle with $A-$Humpty point $H_a$. Let $M$ denote the midpoint of segment $BC$ and $F$ the foot of the altitude from $H_a$ to $BC$. Let $K$ denote the second intersection of $(AH_a)$ and $(ABC)$. Show that circles $(AH_aK)$ and $(KFM)$ are tangent.

The following is the key claim.

Claim : Lines $BK$ and $CH_a$ intersect on $(AH_a)$ (and similarly).

Proof : We show the proof for one as the other is entirely similar. Note,
\[\measuredangle KBC + \measuredangle BCH_a = \measuredangle  KAC + \measuredangle CAH_a = \measuredangle  KAH_a\]which implies that $BK \cap CH_a$ lies on $(AH_a)$ as desired.

Let $X= BK \cap CH_a$ and $Y = CK \cap BH_a$. Now, by Pascal's Theorem on concyclic hexagon $H_aH_aYKKX$ it follows that the tangents to $(AH_a)$ at $K$ and $H_a$ intersect on $\overline{BC}$.

The tangent to $(AH_a)$ at $H_a$ is perpendicular to $AM$ and hence must pass through $H$. Thus, this intersection of tangents is the $A-$Ex point of $\triangle ABC$. Thus, if the tangent intersection point is $T$,
\[TK^2 = TH_a^2 = TF \cdot TM\]which implies that $\overline{TK}$ is also tangent to $(KFM)$ and the result follows.
Z K Y
The post below has been deleted. Click to close.
This post has been deleted. Click here to see post.
ihategeo_1969
285 posts
#149
Y by
Changing labels.
Quote:
Let $\triangle ABC$ have orthic triangle $\triangle DEF$, orthocenter $H$, midpoint of $\overline{BC}$ as $M$. Let $Q_A$ be its $A$-Queue point. Let $K=(Q_AH) \cap (ABC)$. If $AB<AC$ and $A,Q_A,K,B,C$ are cyclic in this order; then prove that $(KQ_AH)$ and $(DKM)$ are tangent to each other.
Invert about $H$ swapping $(ABC)$ and $(DEF)$ which means $K^*=\overline{M\infty_{\perp HM}} \cap (DEF)$. Let $N$ be midpoint of $\overline{AH}$. Now $\measuredangle NK^*M=\measuredangle NDM=90^\circ$; which means $\overline{NK^*} \parallel \overline{HM} \perp \overline{AQ_A} \parallel \overline{MK^*}$ which means $K^*A=K^*Q_A$. This implies $\overline{MK^*}$ is tangent to $(AQ_AK^*)$ and invert back and done.
This post has been edited 2 times. Last edited by ihategeo_1969, Jul 11, 2025, 5:23 PM
Z K Y
The post below has been deleted. Click to close.
This post has been deleted. Click here to see post.
LHE96
14 posts
#150
Y by
I think it was too easy for a P3

Synthetic sol
Z K Y
N Quick Reply
G
H
=
a